How Do You Find a Basis of W from Given Vectors in R4?

  • Thread starter Thread starter Dell
  • Start date Start date
  • Tags Tags
    Basis Matrices
Click For Summary
SUMMARY

The discussion focuses on finding a basis for the subspace W in R4 defined by the vectors w1=(1, 0, 1, -1), w2=(2, 1, 2, -3), and w3=(3, 1, 1, -2). The method involves placing these vectors into a matrix and performing row reduction to identify linearly independent vectors. If no rows can be eliminated, all three vectors form a basis for their span, confirming their linear independence.

PREREQUISITES
  • Understanding of linear algebra concepts, specifically vector spaces and bases.
  • Proficiency in matrix operations, including row reduction and Gaussian elimination.
  • Familiarity with the properties of linear independence in vector sets.
  • Knowledge of R4 and its geometric interpretation.
NEXT STEPS
  • Study the process of Gaussian elimination in detail.
  • Learn about the concept of span and how it relates to vector spaces.
  • Explore the implications of linear independence and dependence in higher dimensions.
  • Investigate the use of MATLAB or Python for matrix operations and visualizing vector spaces.
USEFUL FOR

Students and professionals in mathematics, particularly those studying linear algebra, as well as educators teaching vector spaces and matrix operations.

Dell
Messages
555
Reaction score
0
given these 3 vectors

w1=(1 0 1 -1)
w2=(2 1 2 -3)
w3=(3 1 1 -2)

and i am asked to find a basis of W, subspace of R4

to do this am i supposed to put these vectors into a matrix and try elliminate rows, leaving the basis being the rows i could not get rid of??

does it matter if i put them in a "standing" matrix "A" or a "lying down" matrix "At "

i tried this with an At like matrix, but couldn't elliminate any of them,? are they all 3 the basis or have i done something wrong?
 
Physics news on Phys.org
If you can't create a row of zeros by elimination, then they are linearly independent and you need them all to form a basis of their span, yes.
 
Question: A clock's minute hand has length 4 and its hour hand has length 3. What is the distance between the tips at the moment when it is increasing most rapidly?(Putnam Exam Question) Answer: Making assumption that both the hands moves at constant angular velocities, the answer is ## \sqrt{7} .## But don't you think this assumption is somewhat doubtful and wrong?

Similar threads

Replies
4
Views
2K
  • · Replies 14 ·
Replies
14
Views
3K
Replies
8
Views
2K
  • · Replies 1 ·
Replies
1
Views
2K
  • · Replies 4 ·
Replies
4
Views
2K
  • · Replies 1 ·
Replies
1
Views
2K
  • · Replies 6 ·
Replies
6
Views
2K
  • · Replies 9 ·
Replies
9
Views
3K
  • · Replies 2 ·
Replies
2
Views
2K
  • · Replies 15 ·
Replies
15
Views
3K